Difference between revisions of "2004 AMC 12A Problems/Problem 22"

(+ spacing, imgs. Somebody should do a readibility check)
m (heh, I had forgotten the initial reason behind my edit)
Line 1: Line 1:
 +
{{duplicate|[[2004 AMC 12A Problems|2004 AMC 12A #22]] and [[2004 AMC 10A Problems/Problem 25|2004 AMC 10A #25]]}}
 +
 
== Problem ==
 
== Problem ==
 
Three mutually [[tangent]] [[sphere]]s of [[radius]] <math>1</math> rest on a horizontal [[plane]]. A sphere of radius <math>2</math> rests on them. What is the [[distance]] from the plane to the top of the larger sphere?
 
Three mutually [[tangent]] [[sphere]]s of [[radius]] <math>1</math> rest on a horizontal [[plane]]. A sphere of radius <math>2</math> rests on them. What is the [[distance]] from the plane to the top of the larger sphere?
Line 15: Line 17:
 
== See also ==  
 
== See also ==  
 
{{AMC12 box|year=2004|ab=A|num-b=21|num-a=23}}
 
{{AMC12 box|year=2004|ab=A|num-b=21|num-a=23}}
 +
{{AMC10 box|year=2004|ab=A|num-b=24|after=Final Question}}
  
[[Category:Intermediate Geometry Problems]]
+
[[Category:Introductory Geometry Problems]]

Revision as of 17:22, 4 December 2007

The following problem is from both the 2004 AMC 12A #22 and 2004 AMC 10A #25, so both problems redirect to this page.

Problem

Three mutually tangent spheres of radius $1$ rest on a horizontal plane. A sphere of radius $2$ rests on them. What is the distance from the plane to the top of the larger sphere?

$\text {(A)}\ 3 + \frac {\sqrt {30}}{2} \qquad \text {(B)}\ 3 + \frac {\sqrt {69}}{3} \qquad \text {(C)}\ 3 + \frac {\sqrt {123}}{4}\qquad \text {(D)}\ \frac {52}{9}\qquad \text {(E)}\ 3 + 2\sqrt2$

Solution

2004 AMC12A-22a.png

The height from the center of the bottom sphere to the plane is $1$, and from the center of the top sphere to the tip is $2$. We now need the vertical height of the centers. If we connect the centers, we get a triangular pyramid with an equilateral triangle base. The distance from the vertex of the equilateral triangle to its centroid can be found by $30-60-90 \triangle$s to be $\frac{2}{\sqrt{3}}$.

2004 AMC12A-22b.png

By the Pythagorean Theorem, we have $\left(\frac{2}{\sqrt{3}}\right)^2 + h^2 = 3^2 \Longrightarrow h = \frac{\sqrt{69}}{3}$. Adding the heights up, we get $\frac{\sqrt{69}}{3} + 1 + 2 = \frac{\sqrt{69} + 9}{3} \Rightarrow \mathrm{(B)}$.

See also

2004 AMC 12A (ProblemsAnswer KeyResources)
Preceded by
Problem 21
Followed by
Problem 23
1 2 3 4 5 6 7 8 9 10 11 12 13 14 15 16 17 18 19 20 21 22 23 24 25
All AMC 12 Problems and Solutions
2004 AMC 10A (ProblemsAnswer KeyResources)
Preceded by
Problem 24
Followed by
Final Question
1 2 3 4 5 6 7 8 9 10 11 12 13 14 15 16 17 18 19 20 21 22 23 24 25
All AMC 10 Problems and Solutions